Derivative of directional vector

  • Thread starter Thread starter yecko
  • Start date Start date
  • Tags Tags
    Derivative Vector
Click For Summary
The discussion focuses on finding unit vectors for functions that increase and decrease most rapidly at a point P0, and calculating the directional derivatives. The original poster mistakenly believes the directional derivative is √3, but it is actually 2√3 due to the gradient ∇f being <2,2,2>, which has a length of 2√3. Participants clarify that the OP should verify their gradient calculations to understand the correct directional derivative. The conversation emphasizes the importance of accurate computation in determining the behavior of functions in specified directions.
yecko
Gold Member
Messages
275
Reaction score
15

Homework Statement



Find the unit vectors along which the given functions below increase and decrease most rapidly at P0 . Then find the derivatives of the functions in these directions.

螢幕快照 2017-10-21 下午8.28.59.png


Homework Equations


solution:
螢幕快照 2017-10-21 下午8.24.25.png


The Attempt at a Solution


why are the derivatives’ values along these two directions are 2*sqrt(3)?
as (∇f)(P0)= <1,1,1>, absolute value of it = sqrt(3)
why is the solution multiplied 2 for it?
thanks
 

Attachments

  • 螢幕快照 2017-10-21 下午8.24.25.png
    螢幕快照 2017-10-21 下午8.24.25.png
    8.3 KB · Views: 753
  • 螢幕快照 2017-10-21 下午8.28.59.png
    螢幕快照 2017-10-21 下午8.28.59.png
    1.7 KB · Views: 788
Physics news on Phys.org
yecko said:
as (∇f)(P0)= <1,1,1>,
It isn’t. Double check your computations.
 
yecko said:

Homework Statement



Find the unit vectors along which the given functions below increase and decrease most rapidly at P0 . Then find the derivatives of the functions in these directions.

View attachment 213475

Homework Equations


solution:
View attachment 213474

The Attempt at a Solution


why are the derivatives’ values along these two directions are 2*sqrt(3)?
as (∇f)(P0)= <1,1,1>, absolute value of it = sqrt(3)
why is the solution multiplied 2 for it?
thanks

You are correct: the directional derivative is, indeed, ##2 \sqrt{3}##. The reason is that ##\nabla f(1,1,1) = \langle 2,2,2 \rangle## is a vector of length ##\sqrt{3 \times 4} = 2 \sqrt{3}##.
 
Ray Vickson said:
You are correct: the directional derivative is, indeed, ##2 \sqrt{3}##.
The OP thinks the directional derivative is ##\sqrt{3}## and ##\nabla f = \langle 1,1,1\rangle##, so he is not correct. This is why I suggested him to check his computation of the gradient in #2.
 
o yes... thanks
 
Question: A clock's minute hand has length 4 and its hour hand has length 3. What is the distance between the tips at the moment when it is increasing most rapidly?(Putnam Exam Question) Answer: Making assumption that both the hands moves at constant angular velocities, the answer is ## \sqrt{7} .## But don't you think this assumption is somewhat doubtful and wrong?

Similar threads

Replies
9
Views
2K
  • · Replies 4 ·
Replies
4
Views
1K
  • · Replies 8 ·
Replies
8
Views
2K
Replies
7
Views
2K
  • · Replies 4 ·
Replies
4
Views
2K
  • · Replies 6 ·
Replies
6
Views
2K
  • · Replies 7 ·
Replies
7
Views
6K
  • · Replies 2 ·
Replies
2
Views
2K
  • · Replies 8 ·
Replies
8
Views
5K
Replies
12
Views
9K